PT71.S3.Q26 - as advances in medical research

nathanieljschwartznathanieljschwartz Alum Member
edited August 2017 in Logical Reasoning 1723 karma

Can someone elaborate on why AC D is incorrect?
https://7sage.com/lsat_explanations/lsat-71-section-3-question-26/

Comments

  • OlamHafuchOlamHafuch Alum Member
    2326 karma

    The stimulus doesn't say anything about population increasing in the past few decades. Now, I could grant you that that might be a common sense assumption, but you still don't know anything about the degree of the population increase, or whether its been steady. So, how does D explain why there has been a steady and significant increase in the rate of infection?

  • nathanieljschwartznathanieljschwartz Alum Member
    1723 karma

    But we are trying to resolve a discrepency so it doesnt matter if population increase is mentioned. But since people are living longer it does stand to reason there are more people occupying the world i.e. population increase. And since we are bothered by more infection the AC accounts for this by stating whwn you have a larger population you have more infection?

  • OlamHafuchOlamHafuch Alum Member
    2326 karma

    For (D) to work, you'd have to have a steady and significant increase in the population. You can't just assume that.

  • LSATcantwinLSATcantwin Alum Member Sage
    edited August 2017 13286 karma

    I think this might help a bit.

    what's the actual discrepancy?

    The discrepancy is that health is getting better but infection is on the rise.

    If population increases there is a directly proportional increase in infections. This does nothing to address how health is better yet infection is up. It just says more people are alive so more are sick.

    Do you see how that doesn't address the discrepancy enough?

    If the population is 10 people and 5 are sick

    And now the population is 20 people and 10 are sick

    So what! How are they healthy yet sick at the same time?

Sign In or Register to comment.